Psychopharm MSN 634

Pataasin ang iyong marka sa homework at exams ngayon gamit ang Quizwiz!

TEST 2

TEST 2

A 51-year-old male veteran with chronic posttraumatic stress disorder (PTSD) has agreed to begin pharmacotherapy for his debilitating symptoms of arousal and anxiety associated with his experiences in Iraq. Which of the following would be appropriate as a first-line treatment? Select one: a. Paroxetine b. Paroxetine or diazepam c. Paroxetine, diazepam, or d-cycloserine d. Paroxetine, diazepam, d-cycloserine, or quetiapine

A. Paroxetine, -a SSRI, is the only correct choice. None of the others are approved as first-line treatment for PTSD

Which of the following statements about the heritability of major depressive disorder MDD) is true? Select one: a. The heritability is approximately 40% and the personality trait of neuroticism accounts for a substantial portion of this genetic liability b. Less than 10% of people with genetic liability can be accounted for by the personality trait of perfectionism c. The heritability of MDD depends on whether the individual's mother or father had MDD d. Nearly 100% of people with genetic liability can be accounted for by the personality trait of dogmatism e. Less than 10% of people with genetic liability can be accounted for by the personality trait of perfectionism

A.The heritability is approximately 40% and the personality trait of neuroticism accounts for a substantial portion of this genetic liability First-degree family members of individuals with major depressive disorder have a risk of major disorder two- to fourfold higher than that of the general population. Relative risks appear to be higher for early-onset and recurrent forms. Heritability is approximately 40%, and the personality trait neuroticism accounts for a substantial portion of this genetic liability. Neuroticism (negative affectivity) is a well-established risk factor for the onset of major depressive disorder, and high levels appear to render individuals more likely to develop depressive episodes in response to stressful life events. (166)

Which of the following features distinguishes disruptive mood dysregulation disorder (DMDD) from bipolar disorder in children? Select one: a. Severity b. Age at onset c. Chronicity d. Gender of the child e. Irritability

C. Chronicity -The core feature of DMDD is chronic, severe, persistent irritability. This severe irritability has two prominent clinical manifestations, the first of which is frequent temper outbursts. These outbursts typically occur in response to frustration and can be verbal or behavioral (the latter in the form of aggression against property, self, or others). The clinical presentation of DMDD must be carefully distinguished from presentations of other related conditions, particularly pediatric bipolar disorder. DMDD was added to DSM-5 to address the considerable concern about the appropriate classification and treatment of children who present with chronic persistent irritability relative to children who present with classic (i.e. episodic) bipolar disorder. In DSM-5, the term bipolar disorder is explicitly reserved for episodic presentations of bipolar symptoms. DSM-IV did not include a diagnosis designed to capture youths whose hallmark symptoms consisted of very severe, non-episodic irritability, whereas DSM-5, with the inclusion of DMDD, provides a distinct category for such presentations. (156-157)

A 39-year old woman reports that she became quite depressed in the winter last year when her company closed for the season, but she felt completely normal in the spring. She recalls experiencing several other episodes of depression over the past 5 years (for which she cannot identify a seasonal pattern) that would have met criteria for major depressive disorder. Which of the following correctly summarizes this patient's eligibility for a diagnosis of 'major depressive disorder, with seasonal pattern'? Select one: a. She does qualify for the diagnosis: the single episode started in the winter and ended in the spring. b. She does qualify for this diagnosis: the symptoms described are related to psychosocial stressors c. She does not qualify for this diagnosis: the episode must start in the fall, and the patient must have no episodes that do not have a seasonal pattern d. She does qualify for this diagnosis: the symptoms are not related to bipolar I or bipolar II disorder. e. She does not qualify for this diagnosis: the patient must have had two episodes with a seasonal relationship in the past 2 years and no non-seasonal episodes during that period

E. She does not qualify for this diagnosis: the patient must have had two episodes with a seasonal relationship in the past 2 years and no non-seasonal episodes during that period -The 'with seasonal pattern' specifier requires a regular temporal relationship between the onset of major depressive episodes (MDEs) in major depressive disorder or in bipolar I or bipolar II disorder and a particular time of the year (e.g., in the fall or winter. The diagnosis excludes cases in which there is an obvious effect of seasonal-related psychosocial stressors (e.g., regularly being unemployed every winter.) Full remissions (or a change from major depression to mania or hypomania) also occur at a characteristic time of the years (e.g., depression disappears in the spring). In the past 2 years, two MDEs must have occurred that demonstrate the temporal seasonal relationships defined above, and no non-seasonal MDEs must have occurred during that same time. Seasonal MDEs must substantially outnumber the non-seasonal MDEs that may have occurred over the individual's lifetime. The specifier 'with seasonal pattern' can be applied to the pattern of MDEs in bipolar I disorder, bipolar II disorder, or major depressive disorder, recurrent.

YOU WILL DO GREAT

YOU WILL DO GREAT

A 65-year-old woman reports being housebound despite feeling physically healthy. Several years ago she fell while shopping; although she sustained no injuries, the situation was so upsetting that she became extremely nervous when she had to leave her house unaccompanied. Because she has no children and few friends whom she can ask to accompany her, she is very distressed that she has few opportunities to venture outside her home. What is the most likely diagnosis? Select one: a. Agoraphobia b. Adjustment disorder c. Specific phobia situational type d. Social anxiety disorder (social phobia) e. Post-traumatic stress disorder

a. Agoraphobia -The essential feature of agoraphobia is marked fear or anxiety triggered by real or anticipated exposure to a variety of situations (e.g., using public transportation, going to open or public spaces) from which escape or help might not be available. DSM-IV treated agoraphobia as a feature of panic disorder, and individuals do frequently report a fear of having a panic attack in the dreaded situation; however, there are other incapacitation situations that could cause a similar fear, including fear of falling or of incontinence. This disorder can be very similar to other phobias such as social anxiety disorder and specific phobia, situational type; however the focus of the fear is not the situation itself, but rather the fear that an incapacitating event may take place during the situation. Agoraphobia does not have the cluster of symptoms associated with posttraumatic stress disorder and is not merely indicative of poor adjustment o a uniquely stressful situation. (217-219)

A 19-year-old female requires a hospital consultation after she was admitted following an attempted suicide. Her family reports that she has been suffering from severe depression for the last three years. What area of the brain may best be described as hypothetically involved in her feelings of worthlessness and suicidality? Select one: a. Amydagla b. Nucleus Accumbens c. Basal forebrain d. Hypothalamus

a. Amydagla -The region most associated with feelings of worthlessness and suicidal thoughts is the amygdala

Which of the following is more suggestive of anxiety that is not pathological than of anxiety that qualifies for a diagnosis of generalized anxiety disorder? Select one: a. Anxiety and worry in response to a clear precipitant b. Anxiety and worry focused on a wide range of life circumstances c. Anxiety and worry that interferes significantly with functioning d. Anxiety and worry that lasts for months to years e. Anxiety and worry accompanied by physical symptoms

a. Anxiety and worry in response to a clear precipitant -Several features distinguish generalized anxiety disorder from anxiety that is not pathological. First, the worries associated with generalized anxiety disorder are more pervasive, pronounced, and distressing; have longer duration; and frequently occur without precipitants. Second, the worries associated with generalized anxiety disorder are excessive and typically interfere significantly with psychosocial functioning, whereas the worries of everyday life are not excessive and are perceived as more manageable and may be put off when more pressing matters arise. The greater the range of life circumstances about which a person worries (e.g., finances, children's safety, job performance), the more likely his or her symptoms are to meet criteria for generalized anxiety disorder. Third, everyday worries are much less likely to be accompanied by physical symptoms (e.g., restlessness or feeling keyed up or on edge). Individuals with generalized anxiety disorder report subjective distress due to constant worry and related impairment in social, occupational, or other important areas of functioning. (222-223)

A 24 year-old man with moderate depression achieves remission after sixteen weeks on a therapeutic dose of an antidepressant. According to the neurotrophic hypothesis of depression, which of the following is most likely true of his brain-derived neurotrophic factor (BDNF) expression before and after his successful treatment? Select one: a. BDNF expression was abnormally low while he was depressed, and increased during medication treatment b. BDNF expression was abnormally high while he was depressed, and decreased during medication treatment c. BDNF expression was normal while he was depressed, and was not affected during medication management d. BDNF expression was normal while he was depressed, but suppressed during medication management

a. BDNF expression was abnormally low while he was depressed, and increased during medication treatment -The downstream signal transduction cascade initiated by the use of an antidepressant to treat depression leads to an increased production of BDNF, which while low in depression, can improve depression when at normal levels.

Which of the following antidepressants has the most seizurogenic potential? Select one: a. Buproprion b. Paroxetine c. Venlafaxine d. Mirtazapine

a. Buproprion Buproprion is contraindicated in patient with seizure disorder. The other choices can lower seizure threshhold and so also should be used with caution in a a patient with a history of seizure disorder

A 22-year-old client is taking olanzapine for schizophrenia and complains of difficulty waking up in the morning. What is the most likely cause of this symptom? Select one: a. Drug 'hangover' b. Drug accumulation (toxicity) c. Loss of sleep maintenance d. Comorbid sleep disorder

a. Drug 'hangover' -Olanzapine can be considered a hypnotic with a moderately long half-life (15-30 hours); effects often do not wear off until after time to awaken and therefore a drug 'hangover' can be experienced.

In social anxiety disorder, the object of an individual's fear is the potential for which of the following? Select one: a. Embarrassment b. Separation from objects of attachment c. Incapacitating symptoms d. Harm to self or others e. Social or occupational impairment

a. Embarrassment -The anxiety disorders differ in the object or cause of an individual's fears. In the case of social anxiety disorder, an individual experiences fear or anxiety in situations in which he or she is exposed to scrutiny; the fear is that this may result in humiliation, embarrassment, or offense to others. In contrast, individuals with specific phobia fear harmful objects, animals, or situations; individuals with separation anxiety fear being away from home or loved ones; and individuals with agoraphobia avoid situations in which the individual might have panic-like or other incapacitating symptoms. In all cases, the disorders cause significant social or occupational impairment; however, this impairment is the result rather than the object of the fear.

Which of the following statements about how grief differs from a major depressive episode (MDE) is false? Select one: a. In grief feelings of worthlessness and self-loathing are common; in MDE, self-esteem is generally preserved b. The thought content associated with grief generally features a preoccupation with thoughts and memories of the deceased rather than the self-critical or pessimistic ruminations seen in MDE c. The pain of grief may be accompanied by positive emotions and humor that are uncharacteristic of the pervasive unhappiness and misery characteristic of MDE d. If a bereaved individual thinks about death and dying, such thoughts are generally focused on the deceased person and possibly 'joining' the deceased, whereas in MDE such thoughts are focused on ending one's own life because feeling worthless, undeserving of life, or unable to cope with the pain of depression. e. In grief the predominant affect is feelings of emptiness and loss, while in MDE it is persistent depressed mood and the inability to anticipate happiness or pleasure.

a. In grief feelings of worthlessness and self-loathing are common; in MDE, self-esteem is generally preserved In distinguishing grief from MDE, it is useful to consider that in grief the predominant affect is feelings of emptiness and loss, while in MDE it is persistent depressed mood and the inability to anticipate happiness or pleasure. The dysphoria in grief is likely to decrease in intensity over days to weeks and occurs in waves, the so-called pangs of grief. These waves tend to be associated with thoughts or reminders of the deceased. The depressed mood of MDE is more persistent and not tied to specific thoughts or preoccupations. The pain of grief may be accompanied by positive emotions and humor that are uncharacteristic of MDE. The thought content associated with grief generally features a preoccupation with thoughts and memories of the deceased rather than the self-critical or pessimistic ruminations seen in MDE. In grief, self-esteem is generally preserved, whereas in MDE feelings of worthless and self-loathing are common. If self-derogatory ideation is present in grief, it typically involves perceived failings vis-à-vis the deceased (e.g., not visiting frequently enough, not telling the deceased how much he or she was loved.) If a bereaved individual thinks about death and dying, such thoughts are generally focused on the deceased and possibly about 'joining' the deceased, whereas in MDE such thoughts are focused on ending one's own life because of feeling worthless, undeserving of life, or unable to cope with the pain of depression. (161)

A 65 year-old patient on theophylline for chronic obstructive pulmonary disease (COPD) and fluvoxamine for recurring depressive episodes required a decreased dose of theophylline due to increased blood levels of the drug. Which of the following pharmacokinetic properties of fluvoxamine might be responsible for this? Select one: a. Inhibition of CYP450 1A2 b. Induction of CYP450 1A2 c. Inhibition of CYP 450 2D6 d. Induction of CYP 450 3A4

a. Inhibition of CYP450 1A2 -Fluvoxamine is a strong inhibitor of CYP 450 1A2 which metabolizes theophylline. Thus fluvoxamine can lead to increased serum levels of theophylline, requiring a reduced dose of theophylline

Class of medications that block the action of enzymes that biodegrade the action of the catecholamine neurotransmitters Select one: a. Monoamine Oxidase Inhibitors b. Serotonin Reuptake Inhibitors c. Serotonin Antagonists d. SNRIs

a. Monoamine Oxidase Inhibitors Monoamine oxidase is the enzyme that breaks down the catecholamine neurotransmittters. The class of medications Monoamine Oxidase Inhibitors blocks the action of that enzyme, thereby increasing the presence of the catecholamine neurotransmitters.

A 75-year-old man in good physical shape is having sleep problems. He wakes up at 4 a.m. and although he tries to stay awake in the evening to prevent this early rising, he usually falls asleep right after dinner, often before 7 p.m. Which of the following treatment options may be most beneficial for this man? Select one: a. Morning melatonin and late afternoon / evening light b. Evening melantonin and late afternoon / evening light c. Evening melatonin and early morning light d. Morning melatonin and early morning light

a. Morning melatonin and late afternoon / evening light -This man is 'phase advanced' in his circadian rhythms and may benefit from early morning melatonin and late afternoon / evening light to help reset the suprachiasmic nucleus so that sleep/wake switch stays off longer.

Theory based on concept that the neuronal system is dysfunctional either through too many or too few receptors thus needing to up- or down-regulate the number of receptors Select one: a. Neurogenic Theory of Depression b. Monoamine Theory of Depression c. Glutamate Theory of Depression d. Stress Diathesis Model of Depression

a. Neurogenic Theory of Depression -One theory of how antidepressants relieve depression is the neurogenic theory wherein there is an up and/or down regulation of monoamine receptors

A 45-year-old woman presents with hand-washing compulsion and an obsession with air fresheners. Based on these symptoms, which of the following is most likely to be true? Select one: a. She may have been born with the catechol-O-methyltransferase (COMT) Met genotype, leading to an increased risk of susceptibility to worry, and/or the -s variant of the gene for SERT leading to an increased risk for developing a mood or anxiety disorder b. She may have been born with the catechol-O-methyltransferase (COMT) Met genotype, leading to an increased risk of susceptibility to worry, and/or the -l variant of the gene for SERT leading to an increased risk for developing a mood or anxiety disorder c. She may have been born with the catechol-O-methyltransferase (COMT) Val genotype, leading to an increased risk of susceptibility to worry, and/or the -l variant of the gene for SERT leading to an increased risk for developing a mood or anxiety disorder d. She may have been born with the catechol-O-methyltransferase (COMT) Val genotype, leading to an increased risk of susceptibility to worry, and/or the -s variant of the gene for SERT leading to an decreased risk for developing a mood or anxiety disorder

a. She may have been born with the catechol-O-methyltransferase (COMT) Met genotype, leading to an increased risk of susceptibility to worry, and/or the -s variant of the gene for SERT leading to an increased risk for developing a mood or anxiety disorder -Those born with the COMT Met genotype, as well as those born with the s-variant of the SERT gene may be at increased risk for worry and anxiety disorders. Those born without the METvariant of the COMT genotype and the l-variant of the gene for SERT are at a decreased risk for worry and anxiety.

Social anxiety disorder differs from normative shyness in that the disorder leads to which of the following? Select one: a. Social or occupational dysfunction b. Avoidance of social situations c. Pervasive social deficits with poor insight d. Derealization or depersonalization e. Marked social reticence

a. Social or occupational dysfunction -Shyness is a common personality trait and is not by itself pathological. In some societies, shyness is even evaluated positively. However, when there is a significant adverse impact or social, occupational, and other important areas of functioning, a diagnosis of social anxiety disoarder should be considered, and when full diagnostic criteria for social anxiety disorder are met, the disorder should be diagnosed. Only a minority (12%) of self-identified shy individuals in the United States have symptoms that meet diagnostic criteria for social anxiety disorder (206).

A 35-year-old woman has just been diagnosed with major depressive disorder. For the past 8 months she has had a depressed mood, decreased energy and concentration, and loss of interest in previously enjoyed activities. Although she never attempted suicide, she acknowledges that she thought she would probably jump off a local bridge if she ever had the chance. She denies any history of excessively elevated moods. You decide to start her on antidepressant therapy. Two weeks later this patient is at greatest risk for Select one: a. Suicide completion b. Extrapyramidal symptoms c. Manic episode d. Hypomanic episode e. Medication non-compliance

a. Suicide completion -When antidepressant medications first begin to work, patients tend to report an increase in energy levels before significant improvement in mood symptoms. For this reason, carrying out suicide plans is more of a risk during this period. Medication noncompliance with effective psychopharmacological treatments during both acute and maintenance therapy is a major cause of morbidity among patients with schizophrenia and disorders with poor insight, not among patients with depression. Although noncompliance is a possibility with any patient, it is not the best choice listed. This patient has nothing in her history to indicate bipolar disorder as a more appropriate diagnosis; therefore, there is no reason to believe manic or hypomanic episodes will be precipitated by antidepressant use. Extrapyramidal symptoms are a consequence of typical antipsychotic use, not antidepressant use.

Which of the following statements about gender differences in suicide risk and suicide rates in major depressive disorder (MDD) is true? Select one: a. The risk of suicide attempts is higher for women, but the risk of suicide completions is lower. b. The risk of suicide attempts and completions is higher for women c. The risk of suicide attempts and completions is equal for men and women d. The disparity in suicide rates by gender is much greater in individuals with MDD than in the general population

a. The risk of suicide attempts is higher for women, but the risk of suicide completions is lower. In women, the risk of suicide attempts is higher, and the risk of suicide completions is lower. The disparity in suicide rates by gender is not as great among those with depressive disorders as it is in the population as a whole. (167)

Alpha 1 adrenergic antagonists are medications that block alpha 1 adrenergic receptors. Examples are propranolol and prazosin, which are used 'off label' to treat anxiety, especially stage fright and nightmares Select one: a. True b. Flase

a. True -Alpha 1 adrenergic antagonists / blockers are used 'off label' to treat manifestations of anxiety such as stage fright and nightmares

What is the appropriate method of confirming a diagnosis of premenstrual dysphoric disorder? Select one: a. Two or more months of prospective symptom ratings on validated scales b. Family history c. Neuropsychological testing d. One month of scoring high on the Daily Rating of Severity of Problems or 1 month of scoring high on the Visual Analogue Scales for Premenstrual Mood Symptoms. e. Laboratory tests

a. Two or more months of prospective symptom ratings on validated scales -The diagnosis of premenstrual dysphoric disorder is appropriately confirmed by 2 months of prospective symptom ratings. (Note: The diagnosis may be made provisionally prior to this confirmation.) A number of scales, including the Daily Rating of Severity of Problems or the Visual Analogue Scales for Premenstrual Mood Symptoms have undergone validation and are commonly used in clinical trials for premenstrual dysphoric disorder. The Premenstrual Tension Syndrome Rating Scale has a self-report and an observer version, both of which have been validated and used widely to measure illness severity in women who have premenstrual dysphoric disorder. (173-174)

Children with disruptive mood dysregulation disorder are most likely to develop which of the following disorders in adulthood? Select one: a. Unipolar disorders b. Bipolar II disorder c. Schizophrenia d. Bipolar I disorder e. Borderline personality disorder

a. Unipolar disorders -Approximately half of children with severe, chronic irritability will have a presentation that continues to meet criteria for the condition 1 year later. Rates of conversion from severe, non-episodic irritability to bipolar disorder are very low. Instead, children with chronic irritability are at risk to develop unipolar depressive and/or anxiety disorders in adulthood. (157)

The diagnostic criteria for disruptive mood dysregulation disorder (DMDD) states that the diagnosis should not be made for the first time before age 6 years or after 18 years (Criterion G). Which of the following statements best describes the rationale for this age range restriction? Select one: a. Validity of the diagnosis has been established only in the 7-18 year-old age group b. The restriction is based on existing genetic data c. The restriction represents an attempt to differentiate DMDD from intermittent explosive disorder d. The restriction represents an attempt to differentiate DMDD from bipolar disorder e. The restriction represents an attempt to differentiate DMDD from Oppositional defiant disorder

a. Validity of the diagnosis has been established only in the 7-18 year-old age group -By definition, the onset of DMDD (by history or observation) must be before age 10 years, and the diagnosis should not be applied to children with a developmental level of less than 6 years. It is unknown whether the condition presents only in this age-delimited fashion. Because the symptoms of DMDD are likely to change as children mature, the use of the diagnosis should be restricted to age groups similar to those in which validity has been established (7-18 years). Approximately half of children with severe, chronic irritability will have a presentation that continues to meet criteria for the condition 1 year later. Rates of conversion from severe, non-episodic irritability to bipolar disorder are very low. Instead, children with chronic irritability are at risk to develop unipolar depressive and/or anxiety disorders in adulthood. (157)

Which of the following drugs has data suggesting efficacy in the treatment of menopausal hot flashes Select one: a. Venlafaxine b. Bupropion c. Prazosin d. Clonazepam

a. Venlafaxine -Venlafaxine treatment has been shown to be efficacious in decreasing hot flashes. Clonidine is also used to treat hot sweats but studies concluded that venlafaxine reduces them to a greater extent compared to clonidine

A 32 year-old woman with major depression has been taking a selective serotonin reuptake inhibitor (SSRI) with good response for nine months. She presents now with complaints that she feels numb, and that even when she's sad she can't cry. Her clinician is considering reducing the dose of her SSRI in an effort to alleviate this problem. Is this a reasonable option? Select one: a. Yes, data suggest that SSRI-induced indifference is dose-dependent and can be alleviated by reducing the dose b. No, although data suggest that SSRI-induced indifference is dose-dependent, clients who develop this side effect generally require a switch to a different medication c. No, SSRI-induced indifference is not dose-dependent and thus cannot be alleviated by reducing the dose. d. No, the client's presentation indicates a worsening of depression and the dose of medication should be increased.

a. Yes, data suggest that SSRI-induced indifference is dose-dependent and can be alleviated by reducing the dose -Apathy and emotional blunting can be symptoms of depression, but they are also side effects associated with SSRIs. These symptoms --termed 'SSRI indifference' -- are under-recognized and can be very distressing for patients. Data suggests that lowering the dose can alleviate these 'drug-induced symptoms'.

Circuits of nerves projecting to and from the cortical region of the brain, to the striatum, to the thalamus, and back to the cortical regions that generate and sustain worry are called Select one: a. Nigrostriatal pathway b. Worry circuit c. Fear Circuit d. Suprachiasmic nucleus pathway

b. Worry circuit -The circuit loop from the cortical regions to the striatum and thalamus and back to the cortical region is called the worry circuit, more technically called the cortico-thalamic-striatal-cortical circuit, is hypothesized to generate and sustain worry.

The determination of whether a panic attack is expected or unexpected is ultimately best made by which of the following? Select one: a. The presence or absence of nocturnal panic attacks b. Careful clinical judgment c. 24-hour electroencephalographic monitoring d. Whether the patient associates it with external stress e. Ruling out possible cultural-specific syndromes

b. Careful clinical judgment -It can be difficult to determine whether panic attacks are expected (i.e., triggered by some external stress or situation). Patients (particularly older individuals) may retrospectively attribute panic attacks to certain stressful situations even if they were unexpected in the moment. Laboratory testing may rule out other potential medical causes. Agents with disparate mechanisms of action, such as sodium lactate, caffeine, isoproterenol, yohimbine, carbon dioxide, or cholecystokinin, provoke panic attacks in individuals with panic disorder to a much greater extent than in health control subjects (and in some cases, than in individuals with other anxiety, depressive, or bipolar disorders without panic attacks). In a proportion of individuals with panic disorder, panic attacks are related to hypersensitive medullary carbon dioxide detectors, resulting in hypocapnia and other respiratory irregularities; however, one of these laboratory findings are considered diagnostic of panic disorder. There is no definitive test; ultimately the determination is based on a clinical judgment that takes into account the sequence of events leading to the attack, the patient's own sense of whether triggers are present, and potential cultural factors that may influence a determination of cause.

Although onset of a specific phobia can occur at any age, specific phobia most typically develops during which age period? Select one: a. Old age b. Childhood c. Late adolescence to early adulthood d. Any age e. Middle age

b. Childhood -Specific phobia usually develops in early childhood, with the majority of cases developing prior to age 10 years. The median age at onset is between 7 and 11 years with the mean at about 10 years. Situational specific phobias tend to have a later age at onset than do natural environment, animal, or blood-injection-injury specific phobias. Specific phobias that develop in childhood and adolescence are likely to wax and wane during that period. However, phobias that persist into adulthood are unlikely to remit for the majority of individuals. When specific phobia is being diagnosed in children, two issues should be considered. First, young children may express their fear and anxiety by crying, tantrums, freezing, or clinging. Second, young children typically are not able to understand the concept of avoidance. Therefore, the clinician should assemble additional information from parents, teachers, or others who know the child well. (200).

A 56-year-old male with major depression is brought to the ER with cardiac arrhythmia and possible cardiac arrest. While at the hospital he suffers a seizure. His wife states that he may have ingested an increased dose of his medications. Which of the following is most likely responsible for this apparent overdose reaction? Select one: a. Atomoxetine b. Clomipramine c. Fluoxetine d. Venlafaxine

b. Clomipramine -Clomipramine belongs to the tricyclic class of antidepressants and can cause cardiac arrhythmias at doses in excess of recommended levels.

What DSM-5 diagnostic provision is made for depressive symptoms following the death of a loved one? Select one: a. Depressive symptoms following the loss of a loved one are excluded from receiving a major depressive episode diagnosis; however, a proposed diagnostic category for post-bereavement depression is included in 'Conditions for Further Study (DSM-5 Appendix) pending further research. b. Depressive symptoms following the loss of a loved one are not excluded from receiving a major depressive episode diagnosis if the symptoms otherwise fulfill the diagnostic criteria c. Depressive symptoms lasting less than 2 months after the loss of a loved one are excluded from receiving a diagnosis of major depressive episode. d. To qualify for a diagnosis of major depressive episode, the depressive symptoms in such individuals must include suicidal ideation. e. To qualify for a diagnosis of major depressive episode, the depression must start no less than 12 weeks following the loss

b. Depressive symptoms following the loss of a loved one are not excluded from receiving a major depressive episode diagnosis if the symptoms otherwise fulfill the diagnostic criteria -In DSM-IV there was an exclusion criterion for a major depressive episode that was applied to depressive symptoms lasting less than 2 months following the death of a loved one (i.e., the bereavement exclusion). This exclusion is in DSM-5 for several reasons, including the recognition that bereavement is a severe psychosocial stressor that can precipitate a major depressive episode in a vulnerable individual, generally beginning soon after the loss , and can add an additional risk of suffering, feelings of worthlessness, suicidal ideation, poorer medical health, and worse interpersonal and work functioning. It was critical to remove the implication that bereavement typically lasts only 2 months, when both physicians and grief counselors recognize that the duration is more commonly 1-2 years. A detailed footnote has replaced the more simplistic DSM-IV exclusion to aid clinicians in making the critical distinction between the symptoms characteristic of bereavement and those of major depressive disorder. (811)

A 57-year-old menopausal patient presents complaining of depressed mood, loss of pleasure, widespread painful somatic symptoms with no discernible physical injury, and vasomotor symptoms (VSM). Which of the following options was most recently approved for treating each of these symptoms? Select one: a. Bupropion b. Desvenlafaxine c. Venlafaxine d. Paroxetine

b. Desvenlafaxine Desvenlafaxine is a SNRI that can relieve depressive and menopausal symptoms due to the norepinephrine reuptake inhibition mechanism, Duloxetine does have FDA approval for use in pain managment but has not been shown effective in treatment of vasomotor symptoms.

Benzodiazepines bind to an allosteric site on the GABA receptor causing inhibition to the flow of chloride ions through the channel. Select one: a. True b. False

b. False -Benzodiazepines bind to the allosteric site on the GABA receptor causing the chloride channel to open more frequently, increasing the flow of chloride ions through the channel, thus promoting greater calmness

The peribrachial nucleus, when stimulated by the hippocampus and hypothalamus, is implicated in the fight, flight, freeze syndrome associated with panic Select one: a. True b. False

b. False -The periacqueductal gray area, stimulated by the amygdala, is implicated in the fight, flight, freeze response associated with fear.

A depressed patient reports that he experiences no pleasure from his normally enjoyable activities. Which of the following additional symptoms would be required for this patient to qualify for a diagnosis of major depressive disorder with melancholic features? Select one: a. Despondency, depression that is worse in the morning, and inability to fall asleep b. Inappropriate guilt, depression that is worse in the morning, and early-morning awakening c. Significant weight gain, depression that is worse in the evening, and excessive guilt d. Depression that is worse in the evening, psychomotor agitation, and significant weight loss e. Despondency, significant weight gain, and psychomotor retardation.

b. Inappropriate guilt, depression that is worse in the morning, and early-morning awakening -Two criteria must be met to qualify for the specifier 'with melancholic features' for major depressive disorder. Criterion A specifies that one of the following must be present during the most severe period of the current episode: 1) loss of pleasure in all, or almost all, activities; 2) lack of reactivity to usually pleasurable stimuli (does not feel much better, even temporarily, when something good happens). Criterion B specifies that three (or more) of the following must be present: 1) a distinct quality of depressed mood characterized by profound despondency, despair, and / or moroseness or by so-called empty mood; 2) depression that is regularly worse in the morning; 3) early-morning awakening (at least 2 hours before usual awakening; 4) marked psychomotor agitation or retardation; 5) significant anorexia or weight loss; 6) excessive or inappropriate guilt. The specifier 'with melancholic features' can be applied to the current (or, if full criteria are not currently met for major depressive episode, to the most recent) major depressive episode in major depressive disorder or in bipolar I or II only if it is the most recent type of mood episode.

39-year-old patient with major depressive disorder presents to your office after several months of trying various antidepressant drugs. It is suggested that he try augmenting his current regimen with L-methylfolate (Deplin). Why might L-methyfolate boost the therapeutic efficacy of antidepressants? Select one: a. It enhances the downregulation of post-synaptic autoreceptors b. It both increases the synthesis and inhibits metabolism of catecholamines c. It both decreases the synthesis and enhances the metabolism of monoamines d. It inhibits the downregulation of post-synaptic autoreceptors

b. It both increases the synthesis and inhibits metabolism of catecholamines -L-methylfolate increases the synthesis of the monoamine neurotransmitters and at the same time inhibits the production of COMT (catechol-O-methyltransferase) which breaks down dopamine, inhibiting the metabolism of this monoamine.

The amygdala, activated by fear, connects to this region in the brain that is hypothesized to account for the 'fight, flight, fear' response Select one: a. Nigra striatum b. Periacqueductal gray c. Norepinephrine center d. Hippocampus

b. Periacqueductal gray The periacqueductal gray area, stimulated by the amygdala, is implicated in the fight, flight, freeze response associated with fear.

A 14-year-old boy describes himself as feeling 'down' all of the time for the past year. He remembers feeling better while he was at camp for 4 weeks during the summer; however, the depressed mood returned when he came home. He reports poor concentration, feelings of hopelessness, and low self-esteem, but denies suicidal ideation or changes in his appetite or sleep. What is the most likely diagnosis? Select one: a. Disruptive mood dysregulation disorder b. Persistent depressive disorder (dysthymia) with early onset c. Depressive episodes with short-duration hypomania d. Major disruptive disorder e. Schizoaffective disorder

b. Persistent depressive disorder (dysthymia) with early onset -The essential feature of persistent depressive disorder (dysthymia) is a depressed mood that occurs for most of the day, for more days than not, for at least 2 years, or at least 1 year for children and adolescents. This disorder represents a consolidation of DSM-IV defined chronic major depressive disorder and dysthymic disorder. Major depression may precede persistent depressive disorder, and major depressive episodes may occur during persistent depressive disorder. Individuals whose symptoms meet major depressive disorder criteria for 2 years should be given a diagnosis of persistent depressive disorder as well as major depressive disorder. Individuals with persistent depressive disorder describe their mood as sad or 'down in the dumps.' During periods of depressed mood, at least two of the six symptoms from Criterion B are present. Because these symptoms have become part of the individual's day-to-day experience, particularly in the case of early onset (e.g., 'I've always been this way'), they may not be reported unless the individual is directly prompted. During the 2-year period (1 year for children or adolescents), any symptom-free intervals last no longer than 2 months. (169-170).

A precursor substance from which serotonin is synthesized through enzymatic action in the neuron Select one: a. Glycine b. Tryptophan c. Trazodone d. Tyrosine

b. Tryptophan Tryptophan is the precursor to serotonin

A precursor substance from which both dopamine and norepinephrine are synthesized through enzymatic action Select one: a. Dopamine b. Tyrosine c. Trazodone d. Serotonin

b. Tyrosine Tyrosine is a precursor element necessary for the synthesis of both norepinephrine and dopamine. For serortonin the precursor necessary is tryptophan

A 9 year-old boy cannot go to sleep without having a parent in his room. While falling asleep, he frequently awakens to check that a parent is still there. One parent usually stays until the boy falls asleep. If he wakes up alone during the night, he starts to panic and gets up to find his parents. He also reports frequent nightmares in which he or his parents are harmed. He occasionally calls out that he saw a strange figure peering into his dark room. The parents usually wake in the morning to find the boy asleep on the floor of their room. They once tried to leave him with a relative so they could go on a vacation; however, he became so distressed in anticipation of this that they canceled their plans. What is the most likely diagnosis? Select one: a. Delusional disorder b. Separation anxiety disorder c. Specific phobia d. Agoraphobia e. Nightmare disorder

b. Separation anxiety disorder -The essential feature of separation anxiety disorder is excessive anxiety about being separated from home or attachment figures, beyond what would be expected for the person's developmental stage. By definition, it firsts presents before age 18; however, it may continue into adulthood. Typical presentations include reluctance to leave home or even stay in a room without a parent. In the latter case, children frequently have difficulty at bedtime and may insist that a parent stay with them. They frequently express fear of harm or untoward events that may prevent them from being with a loved one, and they may have nightmares regarding these fears as well as unusual perceptual experiences, particularly at night or in the dark. Although the other disorders should be rule out, the child's focus on a fear of being left alone makes separation anxiety disorder the most likely diagnosis. (191)

A 52-year-old woman with generalized anxiety disorder complains of poor sleep quality and daytime sleepiness. Which treatment plan is most likely to lead to remission? Select one: a. Escitalopram b. Sertraline plus eszopiclone c. Trazodone plus zolpidem d. Estazolam

b. Sertraline plus eszopiclone -Sertraline is an SSRI that is FDA approved for GAD. Eszopiclone is a GABA PAM (positive allosteric modulator) approved for the treatment of insomnia

A 39-year-old male client has been taking sertraline 100mg/day for the last six months to aid in cessation of worry and fear associated with his PTSD. Although still impaired, he has had some response to the drug. Which secondary property of sertraline is most likely to have contributed to the anxiolytic effect? Select one: a. Dopamine transporter inhibition b. Sigma 1 receptor binding c. CYP 2D6 inhibitory properties d. Muscarinic receptor antagonism

b. Sigma 1 receptor binding -Sigma 1 receptor binding of sertraline is thought to contribute to its anxiolytic effects as well as its effects in psychotic and delusional depression. Sertraline's DAT occupancy may contributed to its antidepressant, but not its anxiolyitic effects.

Which of the following most accurately describes people with specific phobias? Select one: a. The fear is usually quite mild in intensity b. The fear occurs almost every time the person encounters the object or situation c. The fear is exactly the same in intensity each time the object or situation is encountered d. Fewer than 10% of people fear more than one object or situation e. The average individual with a phobia has fears of only one object or situation

b. The fear occurs almost every time the person encounters the object or situation -A key feature of specific phobia is that the fear or anxiety is circumscribed to the presence of a particular situation or object (Criterion A), which may be termed the phobic stimulus. Categories of fear situations or objects (i.e., phobic stimuli) are provided as specifiers in the diagnostic criteria. Many individuals fear objects or situations from more than one category. For the diagnosis of specific phobia, the response must differ from normal, transient fears that commonly occur in the population. To meet the criteria for a diagnosis, the fear or anxiety must be intense or severe (i.e., 'marked'; Criterion A). The amount of fear experienced may vary with the proximity to the feared object or situation and may occur in anticipation of or in the actual presence of the object or situation. Also, the fear or anxiety may take the form of a full or limited-symptom panic attack (i.e., expected panic attack). Another characteristic of specific phobias is that the fear or anxiety is evoked nearly every time the individual comes into contact with the phobic stimulus (Criterion B). Thus, an individual who becomes anxious only occasionally upon being confronted with the situation or object (e.g., becomes anxious when flying only on one out of every five airplane flights) would not be diagnosed with specific phobia. However, the degree of fear or anxiety expressed may vary (from anticipatory anxiety to a full panic attack) across different occasions of encountering the phobic object or situation because of various contextual factors, such as the presence of others, duration of exposure, and other threatening elements such as turbulence on a flight for individuals who fear flying. (198-199)

An 18-year-old college student, recently arrived in the United States from Beijing, complains to her gynecologist of irritability, problems with her roommates, increased appetite, feeling bloated, and feeling depressed for 3-4 days prior to the onset of menses. She reports that these symptoms have been present since she reached menarche at age 12 (although she has never kept a mood log). The gynecologist calls you for a consultation about the correct diagnosis, because she is yet unfamiliar with the new DSM-5 diagnostic criteria. What is your response? Select one: a. The patient has no DSM-5 diagnosis b. The patient would qualify for a provisional diagnosis of premenstrual dysphoric disorder c. The patient would qualify for a provisional diagnosis of premenstrual dysphoric disorder; however, the diagnosis does not exist in DSM-5 d. The patient has premenstrual syndrome because she does not meet criteria for premenstrual dysphoric disorder e. The patient would qualify for a provisional diagnosis of premenstrual dysphoric disorder if the diagnosis had been validated in Asian women

b. The patient would qualify for a provisional diagnosis of premenstrual dysphoric disorder -Premenstrual dysphoric disorder is not a culture-bound syndrome and has been observed in individuals in the United States, Europe, India, and Asia. It is unclear as to whether rates differ by race. The essential features of premenstrual dysphoric disorder are the expression of mood lability, irritability, dysphoria, and anxiety symptoms that occur repeatedly during the premenstrual phase of the cycle and remit around the onset of menses or shortly thereafter. These symptoms may be accompanied by behavioral and physical symptoms. Symptoms must have occurred in most of the menstrual cycles during the past year and must have an adverse effect or work or social functioning. Typically symptoms peak around the time of the onset of menses. While the core symptoms include mood and anxiety symptoms, behavioral and somatic symptoms commonly also occur. In order to confirm a provisional diagnosis, daily prospective symptom ratings are required for at least two symptomatic cycles. (172-173)

A 49-year-old man has been taking an MAO inhibitor for the past five years to manage his depression. When he and his wife went out to dinner for their 30th anniversary he had two glasses of Chianti and an Italian meal high in cheese content. He began to experience palpitations, sweating, nausea, vomiting, and extremely elevated blood pressure. His wife rushed him to the ER, where the doctors asked what medications he is currently taking. Which of the following could be responsible for these symptoms? Select one: a. Oral selegiline 10mg/day b. Tranylcypromine 30mg/day c. Fluvoxamine 20mg /day d. Transdermal selegiline patch 6mg/day

b. Tranylcypromine 30mg/day -Tranylcypromine is responsible. The client has eaten a meal presumably with high tryamine content. Clients taking the MAOI inhibitor tranycypromine can experience a hypertensive crisis if they eat a meal high in tyramine content. The oral selegiline is not a therapeutic dose for treating depression and will not cause a hypertensive crisis. The transdermal patch of selegiline bypasses the gastric absorption and thus is not a cause of hypertensive crisis.

A suicidal patient with chronic depressive disorder presents to your office very frustrated and in tears. He tells you he cannot stop thinking about ending his life because he is so depressed. You ask him if he has a plan, and he details where he could buy a handgun and where he would go to shoot himself. You fear the patient will carry out this plan because he has not had adequate control of his symptoms since his last antidepressant change 1 month ago. You discuss inpatient hospitalization for medication stabilization, but the patient refuses. Your next step in management of this patient would best be Select one: a. Give the latest antidepressant more time to take effect b. Initiate psychotherapy to discuss the reasons behind the suicidal thoughts c. Admit the patient to the hospital anyway d. Try to persuade the patient to admit himself to the hospital e. Change to another class of antidepressant

c. Admit the patient to the hospital anyway Suicidal patients with intent and specific plans should always be taken seriously. For patients believed to be too much at risk for outpatient or partial hospital programs, inpatient treatment is required. Such hospitalization is preferably on a voluntary basis, but if the patient refuses, involuntary admission is required. The options of changing antidepressants and giving more time for the medication to take effect are not the best options in this circumstance, although they may be when suicide is less of an acute risk. Psychotherapy is also not an option when immediate intervention is needed with actively suicidal patients. Contact with the family and friends of suicidal patients is essential, and maintaining patient confidentiality is not mandatory if divulged material is believed to be necessary to protect the patient's life. Some patients appear so imminently and acutely suicidal that the clinician is afraid to let them out of the offices. Patients can be admitted to the hospital against their will if they are a danger to themselves or other people.

A 45-year-old woman was prescribed doxepin 10mg / night for insomnia. She reports that it helped only a little so she has been increasing the dose up to 100mg/ night in an attempt to increase the hypnotic effects (with some success). She also reports dizzy spells and constipation. Which property does doxepin exhibit in higher doses that could be the cause of the side effects? Select one: a. Inhibiting reuptake of serotonin and norepinephrine b. 5HT2A and 5HT2C antagonism c. Alpha 1 adrenergic and muscarinic 1 antagonism d. 5HT2A and 5H2B antagonism

c. Alpha 1 adrenergic and muscarinic 1 antagonism -Low dose doxepin (Silenor) is selective for histamine 1 receptors, which is why it can act as a hypnotic. It is likely that the alpha 1 adrenergic and muscarinic 1 receptor antagonism seen with higher doses of doxepin would explain the client's side effects.

A 19-year-old woman presents with social anxiety disorder. She has been experiencing panic attacks when she is in extremely crowded areas, and has begun to avoid such areas altogether. Which of the following GABA-A receptors is most likely involved? Those containing Select one: a. Alpha 1 subunits b. Alpha 2 subunits c. Alpha 2 and or 3 subunits d. Alpha 4 subunits

c. Alpha 2 and or 3 subunits -Alpha 2 and 3 subunits are theoretically involved in anxiety. Alpha 1 subunits are theoretically involved in sleep. Alpha 4 subunits are theoretically involved in tonic inhibition of GABA.

Medications that work by special mechanisms such as dopamine reuptake inhibition or noradrenergic blockage Select one: a. Tryptophan enhancers b. Tricyclic Antidepressants c. Atypical Antidepressants d. First generation Antidepressants

c. Atypical Antidepressants -Atypical antidepressants work by mechanism other than the classic serotonin reuptake inhibition or dual serotonin - norepinephrine reuptake inhibition. They do not fit into the definitions for first generation or tricylic antidepressants

A 31-year-old female assault victim is brought to the emergency room after tracking down passers-by for help. She appears traumatizd from the incident. Which of the following pharmacotherapy options has been theorized as a potential preemptive treatment to the development of PTSD? Select one: a. N-methyl-D-aspartate (NMDA) agonist such as d-cycloserine b. Alpha 2 delta ligand such as pregabalin c. Beta adrenergic blocker such as propanolol d. Benzodiazepine such as diazepam

c. Beta adrenergic blocker such as propanolol -Beta adrenergic blockers have been shown to block formation of fear-conditioning immediately following trauma. Thus, in this case, propranolol may be theoretically useful -- off label -- to aid in decreasing the risk of PTSD due to her incident

A 26-year-old woman presents to your office after experiencing a recent decrease in the effectiveness of codeine, which she has been taking for several months with previously positive results for irritable bowel syndrome accompanied by moderate abdominal pain. However, she was recently prescribed paroxetine. Which of the following properties of paroxetine may be responsible for the decreased efficacy of codeine? Select one: a. Anticholinergic effects (M1 receptor) b. Serotonin transporter (SERT) inhibition c. CYP450 2D6 inhibition d. Nitric Oxidase inhibition

c. CYP450 2D6 inhibition While all options are properties of paroxetine, it is the CYP450 2D6 inhibition by paroxetine that could potentially interfere with the analgesic effect of codeine by reducing its metabolism to morphine, thereby rendering it less efficacious. Nitric oxidase synthethase inhibition may contribute to the sexual dysfunction associated with paroxetine

What is the primary difference in the clinical expression of generalized anxiety disorder across age groups? Select one: a. Predominance of cognitive versus somatic symptoms b. Severity of impairment c. Content of worry d. Degree of worry e. Patterns of comorbidity

c. Content of worry -The clinical expression of generalized anxiety disorder is relatively consistent across the life span and the primary difference across age groups is the content of an individual's worry. Children and adolescents tend to worry about school or sports performance, whereas adults are more likely to be concerned about their personal health or the well-being of their family

A 34-year-old woman with an active lifestyle presents with complaints that she has recently been falling asleep in the middle of the day; on three occasions it has caused her to be nearly an hour late to pick up her 7-year-old son from school. Which transmitters is most likely affected in this woman? Select one: a. Excessive histamine from the tuberomammillary nucleus b. Excessive serotonin to the thalamus c. Deficient dopamine to the nucelus accumbens d. Deficient GABA from the ventrolateral preoptic nucleus

c. Deficient dopamine to the nucelus accumbens -When dopaminergic input to the nucleus accumbens is hypoactive during the day it can enhance the effectiveness of the thalamaic filter, not allowing enough sensory input to reach the cortex, which can result in daytime sleepiness.

A 12-year old boy begins to have new episodes of temper outburst that are out of proportion to the situation. Which of the following is not a diagnostic possibility for this patient? Select one: a. Oppositional defiant disorder b. Bipolar disorder c. Disruptive mood dysregulation disorder d. Conduct disorder e. Conduct disorder

c. Disruptive mood dysregulation disorder Criteria G and H of disruptive mood regulation disorder state that the chronological age at onset is at least 6 years (or equivalent developmental level) and the onset is before 10 years. (156)

A 42-year-old police officer presents with sleep cycle disturbances. After sufficient consultation you prescribe her ezopiclone. Which receptors are primarily being targeted by this treatment? Select one: a. Histamine 1 b. Serotonin 2A c. GABA A alpha 1 isoform

c. GABA A alpha 1 isoform -Ezopiclone is a GABA positive allosteric modulator and targets GABA A alpha 1 isoforms. I

A 44-year-old man taking paroxetine for depression reports experiencing sexual dysfunction. He opts to discontinue pharmacotherapy, at which time he experiences akathisia and dizziness. Which of the following combination of properties might respectively cause the dysfunction and the discontinuation symptoms? Select one: a. Anticholinergic actions and disinhibition of nitric oxide synthetase (NOS) b. Inhibition of CYP 450 3A4 and inhibition of nitic oxide synthetase (NOS) c. Inhibition of nitric oxide synthetase (NOS) and anticholinergic actions d. Inhibition of CYP 450 2D2 and anticholinergic actions

c. Inhibition of nitric oxide synthetase (NOS) and anticholinergic actions -Paroxetine can impair sexual function possibly through inhibition of nitric oxide synthetase and through anticholinergic actions

A 50 year-old man presents with persistently depressed mood for several weeks that interferes with his ability to work. He has insomnia and fatigue, feels guilty, has thoughts he would be better off dead, and has thought about how he could die without anyone knowing it was a suicide. His wife informs that he requests sex several times a day and that she thinks he may be going to 'massage parlors' regularly, both of which are changes from his typical behavior. He has told her he has ideas of a 'better Internet,' and he has invested thousands of dollars in software programs that he cannot use. She notes that he complains of fatigue but sleeps only 1-2 hours each night and seems to have tremendous energy during the day. Which diagnosis best fits this patient? Select one: a. Manic episode b. Hypomanic episode c. Major depressive episode with mixed features d. Major depressive episode with atypical features e. Major depressive episode

c. Major depressive episode with mixed features -The specifier 'with mixed features' now denotes the coexistence of at least three manic symptoms insufficient to satisfy criteria for a manic episode, within a major depressive episode. This change is based on findings from studies of family history and diagnostic stability showing that the presence of mixed features in an episode of major depressive disorder increased the likelihood that the illness exists in a bipolar spectrum. This likelihood was judged insufficient to assign such individuals a diagnosis of bipolar disorder. The presence of full manic syndrome within a depressive episode will continue to be an exclusion criterion for a depressive disorder diagnosis, and individuals with this pattern will be considered to have a manic episode. (184-185)

Which of the following symptoms must be present for a woman to meet criteria for premenstrual dysphoric disorder? Select one: a. A feeling of overwhelmed or out of control b. Physical symptoms such as breast tenderness c. Marked affective lability d. Decreased interest in usual activities e. Marked changes in appetite

c. Marked affective lability -Of the 11 symptoms in the premenstrual dysphoric disorder diagnostic criteria, patients must have a total of at least 5 symptoms. One of the 5 must be one of the following symptoms: 1) marked affective lability; 2) marked irritability or anger or increased interpersonal conflicts; 3) marked depressed mood, feelings of hopelessness, or self-deprecating thoughts; 4) marked anxiety tension, and/or feelings of being keyed up or on edge. (171-172)

In addition to feeling restless or 'keyed up,' individuals with generalized anxiety disorder are most likely to experience which of the following symptoms? Select one: a. Panic attacks b. Obsessions c. Muscle tension d. Social anxiety e. Multiple somatic complaints

c. Muscle tension -Generalized anxiety disorder is defined as excessive anxiety and worry that occurs more days than not, lasts for at least 6 months, and is associated with restlessness or feeling keyed up or on edge and muscle tension. The anxiety cannot be due to other anxiety disorders; the symptoms listed in the other options suggest other disorders that would be part of the differential diagnosis (i.e., panic disorder, obsessive-compulsive disorder, somatic symptom disorder, and social anxiety disorder. (222)

A client whose hypertension is well-controlled on clonidine presents with major depression. Which of the following antidepressants should be avoided in the treatment of his depression? Select one: a. Fluoxetine b. Paroxetine c. Nortriptyline d. Bupropion

c. Nortriptyline Nortriptyline when administered with clonidine may result in hypertension because the combination decreases the effects of clonidine. No drug-to-drug of clonidine has been reported with paroxetine, fluoxetine, or bupropion.

A 35-year-old Army veteran presents with a chief complaint of excessive crying, which he notes is often triggered by thoughts of dying prior to seeing his family again. In addition, he describes being unable to sleep most nights and has difficulty concentrating on the task at hand. He reports that he is often tense and seems to fly off the handle at his wife more than usual, which is often a precursor to bouts of difficulty breathing and increased heart rate. His symptom of difficulty breathing is hypothetically related to activation in the Select one: a. Hippocampus b. Hypothalamus c. Parabrachial nucleus d. Periaqueductal gray

c. Parabrachial nucleus -The parabrachial nucleus (PBN) regulates changes in respiration, which can occur during fear response. This regulation is activated by the amygdala. Excessive activation of the PBM can lead to an increased rate of respiration and symptoms such as shortness of breath or a sense of being smothered.

A 55-year-old man has been taking buspirone for generalized anxiety disorder for the past year. He gladly notes that recently he has begun to experience an increased ability to concentrate and is more interested in participating in soccer games, which he used to thoroughly enjoy. What may be the underlying mechanism of buspirone accountable for this positive response? Select one: a. Initial increase of serotonin b. Decreased glutamate release c. Partial agonist actions at 5HT1A receptors d. Partial antagonist actions at 5HT1B / D receptors

c. Partial agonist actions at 5HT1A receptors -5HT1A partial agonists may reduce anxiety by actions and both presynaptic somadendritc autoreceptors and at postsynaptic receptors. Buspirone does not act on 5HT1B/D receptors

A 26-year-old female treated for depression eight months ago has recently begun to experience a renewal of symptoms - loss of interest, depressed mood, irritability. According to general consensus, her current expression of symptoms could be classified as a Select one: a. Response b. Remission c. Relapse d. Recurrence

c. Relapse -A relapse is determined to occur when symptoms of depression reappear during the remission stage of symptom free for 6 months to a year.

Which of the following characteristics of generalized anxiety disorder is especially common in children who have the disorder? Select one: a. Excessively preparing for activities b. Delaying or procrastinating before activities c. Seeking frequent reassurance from others d. Avoiding activities that may provoke anxiety e. Complaining of physical aches and pains

c. Seeking frequent reassurance from others -All of the behaviors listed are typical of generalized anxiety disorder; however, seeking reassurance from others (i.e., friends, family, or practitioners) is especially common in children. (224)

Which of the following disorders is included in the 'Anxiety Disorders' chapter of DSM-5? Select one: a. Obsessive Compulsive Disorder b. Acute stress disorder c. Separation anxiety disorder d. Post-traumatic stress disorder

c. Separation anxiety disorder -The DSM-5 'Anxiety Disorders' chapter contains a number of additions and deletions when usually first diagnosed in infancy, childhood, or adolescence are now included among the DSM-5 anxiety disorders, including separation anxiety disorder and selective mutism. Several DSM-IV disorders from the 'Anxiety Disorders' chapter, including obsessive compulsive disorder, posttraumatic stress disorder, and acute stress disorder were removed from that section in DSM-5. This reorganization was the result of a scientific review that concluded that these were distinct disorders that were not sufficiently described by the presence of anxiety symptoms. Agoraphobia has been separated from panic disorder as a distinct disorder in DSM-5, which includes a panic attack specifier when they occur.(189-190)

A 36-year-old depressed patient's fMRI results indicate increased activity in the ventromedial prefrontal cortex. Which of the following treatment options is hypothetically best able to prevent excessive release of glutamate from pyramidal neurons and aid in pyramidal cell functioning in PFC areas? Select one: a. Selective serotonin reuptake inhibitors b. Serotonin norepinephrine reuptake inhibitors c. Serotonin antagonist / reuptake inhibitors d. Norepinephrine and dopamine reuptake inhibitors

c. Serotonin antagonist / reuptake inhibitors ~ 5HT antagonism inhibits the action of glutamate in the prefrontal cortex leading to a decreased release of glutamate

A young woman has just been diagnosed with PTSD and is ready to begin medication treatment. Which of the following has the most evidence of efficacy as a first-line pharmacologic treatment in PTSD? Select one: a. Pregabalin b. Duloxetine c. Sertraline d. Alprazolam

c. Sertraline -Sertraline is a first line FDA approved option for PTSD. Alprazolam is contraindicated or discouraged. Duloxetine is not a first line option and neither is pregabalin.

A 35-year-old man is in danger of losing his job because it requires frequent long-range traveling and for the past year he has avoided flying. Two years earlier he was on a particularly turbulent flight, and although he was not in any real danger, he was convinced that the pilot minimized the risk and that the plane almost crashed. He flew again 1 month later, and, despite having a smooth flight, the anticipation of turbulence was so distressing that he experienced a panic attack during the flight. He has not flown since. What is the most appropriate diagnosis? Select one: a. Agoraphobia b. Social anxiety disorder (social phobia) c. Specific phobia, situational type d. Acute stress disorder e. Panic disorder

c. Specific phobia, situational type -Specific phobia is characterized by the marked fear or anxiety of a specific object or situation, which is perceived as being dangerous. This differs from agoraphobia, in which the focus of the anxiety is on the possibility of having panic or other incapacitating symptoms, or social anxiety disorder in which the focus is on being scrutinized by others. Trauma-related disorders should be considered in the differential diagnosis; however, the lack of any real danger makes this unlikely, and the time course is not compatible with the criteria for acute stress disorder. Although the man did experience a panic attack, patients with many disorders, including specific phobia, can experience such attacks. Panic disorder should be diagnosed only when the attacks are unexpected and not otherwise explained by other disorders (197-199)

Which of the following statements about the heritability of major depressive disorder MDD) is true? Select one: a. Nearly 100% of people with genetic liability can be accounted for by the personality trait of dogmatism b. Nearly 50% of people with genetic liability can be accounted for by the personality trait of aggressiveness. c. The heritability is approximately 40% and the personality trait of neuroticism accounts for a substantial portion of this genetic liability d. The heritability of MDD depends on whether the individual's mother or father had MDD e. Less than 10% of people with genetic liability can be accounted for by the personality trait of perfectionism.

c. The heritability is approximately 40% and the personality trait of neuroticism accounts for a substantial portion of this genetic liability First-degree family members of individuals with major depressive disorder have a risk of major disorder two- to fourfold higher than that of the general population. Relative risks appear to be higher for early-onset and recurrent forms. Heritability is approximately 40%, and the personality trait neuroticism accounts for a substantial portion of this genetic liability. Neuroticism (negative affectivity) is a well-established risk factor for the onset of major depressive disorder, and high levels appear to render individuals more likely to develop depressive episodes in response to stressful life events. (166)

A 33-year-old man taking estazolam to help him sleep through the night complains that his symptoms have returned despite his adherence to the medication regimen for the past six months. You suspect he has developed a tolerance to this drug and elect to switch him to another medication. Which would be the more appropriate choice? Select one: a. Temazepam b. Ramelteon c. Trazodone d. Chlordiazepoxide

c. Trazodone -Trazodone would be the more appropriate choice because it works by a serotonergic and histaminergic mechanism which is different from the GABA mechanism of eztazolam. Ramelteon might be a choice, but it is best for sleep initiation, not sleep maintenance. Temazepam and chlordiazepoxide are both GABA agents and since the client has possibly developed a tolerance to this type of mechanism are not good choices.

A 29 year old patient has tried several treatment options to manage his depression, which is characterized by decreased alertness, fatigue, and low self-confidence. With previous treatments he has experienced only partial resolution of these symptoms and has had the intolerable side effect of sexual dysfunction. Following the concept of linking mechanism of action to neurobiology of symptoms, which of the following treatment combinations may be best for this patient? Select one: a. fluoxetine + olanzapine b. escitalopram + eszopiclone c. bupropion + modafinil d. venlafaxine + trazodone

c. bupropion + modafinil -Bupropion may increase mental alertness and modafinil may stimulate wakefulness

A 34 year-old man with depression characterized by depressed mood, sleep difficulties, and concentration problems has not responded well to three sequential antidepressant monotherapies. His provider elects to add an agent with prominent 5HT7 antagonism such as lurasidone (Latuda) or asenapine (Saphris). What may be a primary function of the 5HT7 receptors? Select one: a. regulation of serotonin-acetylcholine interactions b. regulation of serotonin-dopamine interactions c. regulation of serotonin-glutamate interactions d. regulation of serotonin-norepinephrine interactions

c. regulation of serotonin-glutamate interactions

A 26-year-old client with panic disorder is willing to begin pharmacotherapy for treatment of his phobias and panic attacks. Clonazepam is suggested; what would be your recommended dose? Select one: a. 4mg/ day, uptitrating to 6mg/day after 1 week b. 2mg/day uptitrating every 3 days until 20mg/day is reached c. 1.5mg/day, uptitrating 0.5mg every other day until a dose of 10mg / day is reached d. 0.5mg bid for several days, then titrate up by 0.5mg bid with a maximum of 4mg total daily

d. 0.5mg bid for several days, then titrate up by 0.5mg bid with a maximum of 4mg total daily -Clonazepam is usually dosed bid (some sources prefer tid) starting at low dose of 0.5mg and working up to the maximum recommended dose of 4mg total in a day.

A 48 year-old man with depression was recently started on 20mg/day of fluoxetine to combat his presenting symptoms of apathy, fatigue, problems concentrating, and hypersomnia. The gentleman reports he is feeling much more energized and can see improvements in his cognition and attention within a week or two of starting fluoxetine. Which of the following properties of fluoxetine is theoretically most likely responsible for this man's positive response? Select one: a. Serotonin reuptake inhibition b. Downregulation of serotonin autoreceptors c. Norepinephrine reuptake inhibition d. 5HT2C antagonism

d. 5HT2C antagonism Fluoxetine acts as a 5HT2C antagonist, a property unique among the SSRIs. Antagonism of the 5HT2C receptors indirectly increases norepinephrine and dopamine in the prefrontal cortex, improving cognition and attention.

A 38-year-old patient with depression presents with the following symptoms: loss of happiness, loss of interest, fear/anxiety, and irritability. Which of the following describes these symptoms of depression? Select one: a. Increased negative affect b. Serotonergic dysfunction c. Dopaminergic dysfunction d. All of these

d. All of these This client's symptoms can be described by any and all of the presented choices

A site on the GABA receptor that is other than the GABA binding site, when activated, enhances the binding and activity of GABA. Select one: a. GABA interneurons b. GABA isoforms c. Gabapentin reflectors d. Allosteric binding site

d. Allosteric binding site Allosteric binding sites are located on the GABA receptor in a location other than the GABA binding site. Agonist binding at this site is called a positive allosteric modulator (PAM).

Receptors comprised of five isoform regions around a chloride channel are which of the following Select one: a. Gabapentin receptors b. Chloride ion channels c. Positive allosteric modulators d. GABA receptors

d. GABA receptors GABA receptors are composed of five isoform regions, which themselves are each formed from four transmembrane units. The five isoforms surround a chloride ion channel.

A 44 year-old woman with unipolar depression presents with complaints of weight gain over the past months. She has been on 30mg/day of mirtazapine, and acknowledges improved cognition and sleep, but is unhappy with her weight. Which of the following properties might be responsible for this unfavorable side effect? Select one: a. Inhibition of 5HT2A receptors b. Agonist action at 5HT1A receptors and antagonism of histamine receptors c. Inhibition of 5HT2C and muscarinic receptors d. Inhibition of 5HT2C and histamine receptors

d. Inhibition of 5HT2C and histamine receptors Antagonism, inhibition of 5HT2C and histamine receptors is linked to weight gain

The region in the brain stem involved in the synthesis / release of norepinephrine leading to endocrine activation of the HPA axis is the Select one: a. Worry circuit b. Fear circuit c. Amygdala d. Locus cereleus

d. Locus cereleus The locus cereleus is the brain region where norepinephrine is synthesized from tyrosine

A 38-year-old woman with recently diagnosed major depressive disorder is seeking medication treatment. Because she takes multiple other medications, her care provider prefers to select an antidepressant that does not induce or inhibit any CYP450 enzyme. Which of the following treatment options could potentially be utilized? Select one: a. Citalopram b. Fluoxetine c. Sertraline d. Mirtazapine

d. Mirtazapine -Mirtazapine doesn't induce or inhibit any CYP 450 enzymes

Which of the following statements about diagnostic markers for major depressive disorder (MDD) is true? Select one: a. Tests that exist are adequate diagnostically but are not covered by health insurance b. Several diagnostic laboratory tests exist, but no commercial enterprise will offer them to the public c. Only function magnetic resonance imaging (fMRI) provides absolute diagnostic reliability for MDD d. No laboratory test has demonstrated sufficient specificity to be used as a marker for MDD e. Diagnostic laboratory tests have been withheld for fear that people testing positive for MDD may attempt suicide

d. No laboratory test has demonstrated sufficient specificity to be used as a marker for MDD -Although an extensive literature exists describing neuroanatomical, neuroendocrinological, and neurophysiological correlates of MDD, no laboratory tests has yield results of sufficient sensitivity and specificity to be used as a diagnostic tool for this disorder. Until recently, hypothalamic-pituitary-adrenal axis hyperactivity had been the most extensively investigated abnormality associated with major depressive episodes, and it appears to be associated with melancholia, psychotic features, and risks for eventual suicide. Molecular studies have also implicated peripheral factors, including genetic variants in neurotrophic factors and pro-inflammatory cytokines. Additionally, fMRI studies provide evidence for functional abnormalities in specific neural systems supporting emotion processing, reward seeking, and emotion regulation in adults with depression (165).

Which of the following statements best describes how panic attacks differ from panic disorder? Select one: a. Panic attacks require few symptoms for a definitive diagnosis b. Panic attacks are discrete, occur suddenly, and are usually less severe c. Panic attacks are invariably unexpected d. Panic attacks represent a syndrome that can occur with a variety of other disorders e. Panic attacks cannot be secondary to a medical condition

d. Panic attacks represent a syndrome that can occur with a variety of other disorders -Panic attacks are abrupt surges of intense fear or intense discomfort that reach a peak within minutes, accompanied by physical and/or cognitive symptoms. Panic attacks may be either expected (e.g., in response to a typically feared object or situation) or unexpected (meaning that the panic attack occurs for no apparent reasons). Although DSM-5 defines symptoms for the purpose of identifying a panic attack, panic attack is not a mental disorder and cannot be coded. Panic attacks can occur in the context of any anxiety disorder as well as other mental disorders (e.g., depressive disorders, posttraumatic stress disorder, substance use disorders) and some medical conditions (e.g., cardiac, respiratory, vestibular, gastrointestinal). When the presence of a panic attack is identified, it should be noted as a specifier (e.g., 'posttraumatic stress disorder with panic attacks'). For panic disorder, the presence of panic attack is contained within the criteria for the disorder, and panic attack is not used as a specifier. (214-215)

A 30-year-old woman reports 2 years of persistently depressed mood, accompanied by loss of pleasure in all activities, ruminations that she would be better off dead, feelings of guilt about bad things she has done, and thoughts about quitting work because of her inability to make decisions. Although she has never been treated for depression, she feels so distressed at times that she wonders if she should be hospitalized. She experiences an increased need for sleep but still feels fatigued during the day. Her overeating has led to a 24 lb. weight gain. She denies drug or alcohol use, and her medical workup is completely normal, including laboratory tests for vitamins. The consultation was prompted by her worsened mood for the past several weeks. What is the most appropriate diagnosis? Select one: a. Bipolar II disorder b. Major depressive disorder with melancholic features c. Major depressive disorder d. Persistent depressive disorder (dysthymia) with persistent major depressive episode e. Cyclothymia

d. Persistent depressive disorder (dysthymia) with persistent major depressive episode -The essential feature of persistent depressive disorder (dysthymia) is a depressed mood that occurs for most of the day, for more days than not, for at least 2 years, or at least 1 year for children and adolescents. This disorder represents a consolidation of DSM-IV defined chronic major depressive disorder and dysthymic disorder. Major depression may precede persistent depressive disorder, and major depressive episodes may occur during persistent depressive disorder. Individuals whose symptoms meet major depressive disorder criteria for 2 years should be given a diagnosis of persistent depressive disorder as well as major depressive disorder. If there is a depressed mood plus two or more symptoms meeting criteria for a persistent depressive episode for 2 years or more, then the diagnosis of persistent, depressive disorder is made. The diagnosis depends on the 2-year duration, which distinguishes it from episodes of depression that do not last 2 years. If the symptom criteria are sufficient for a diagnosis of a major depressive episode at any time during this period, then the diagnosis of major depression should be noted, but it is coded not as a separate diagnosis, but rather as a specifier with the diagnosis of persistent depressive disorder. If the individual's symptoms currently meet full criteria for a major depressive episode, then the specifier 'with intermittent major depressive episodes, with current episode,' would be applied. If - as in the patient described above - the major depressive episode has lasted for at least a 2-year duration and remains present, then the specifier, 'with persistent major depressive episode' is used. When full major depressive episode criteria are not currently met but there has been at least one previous period of major depression in the context of at least 2 years of persistent depressive symptoms, then the specifier 'with intermittent depressive episodes, without current episode' is used. If the individual has not experienced an episode of major depression in the past two years, then the specifier 'with pure dysthymic syndrome' is used. (168-169)

A 46-year-old female client has been experiencing severe anxiety based symptoms for many years, and was previously diagnosed with generalized anxiety disorder. She describes difficulty concentrating in addition to difficulty falling asleep. Her family has recently told her that she seems to be displaying heightened anger responses toward them over minor details. Oftentimes she will cry for extended periods of time and become irritable and distant. Based on the client's revelations, if she were to continue to experience these stressful reactions to stimuli (i.e. excessive crying, fatigue, problems concentrating, tension, irritability), what could potentially occur? Select one: a. Increased hippocampal volume and increased brain derived neurotrophic factor (BDNF) b. Decreased hippocampal volume and increased BDNF c. Reduced reactivity to stress d. Reduced BDNF and decreased hippocampal volume

d. Reduced BDNF and decreased hippocampal volume -Reduced brain derived neurotrophic factor (BDNF) can occur in people who experience chronic stress, leading to a decreased ability to create and maintain neuronal connections. Decreased hippocampal volume, perhaps related to the expression of BDNF, has been reported in some chronic stress conditions such as depression and certain anxiety disorders

A 45 year-old man with classic features of schizophrenia has always experienced co-occurring symptoms of depression - including feeling 'down in the dumps,' having a poor appetite, feeling hopeless, and suffering from insomnia - during his episodes of active psychosis. These depressive symptoms occurred only during his psychotic episodes and only during the 2-year period when the patient was experiencing active symptoms of schizophrenia. After his psychotic episodes were successfully controlled by medication, no further symptoms of depression were present. The patient has never met full criteria for major depressive disorder at any time. What is the appropriate DSM-5 diagnosis? Select one: a. Persistent depressive disorder (dysthymia) b. Schizoaffective disorder c. Unspecified schizophrenia spectrum and other psychotic disorder d. Schizophrenia e. Schizophrenia and persistent depressive disorder (dysthymia)

d. Schizophrenia Depressive symptoms are a common associated feature of chronic psychotic disorders (e.g. schizoaffective disorder, schizophrenia, delusional disorder). A separate diagnosis of persistent depressive disorder is not made if the symptoms occur only during the course of the psychotic disorder (including residual phase). (171)

A 39-year old woman reports that she became quite depressed in the winter last year when her company closed for the season, but she felt completely normal in the spring. She recalls experiencing several other episodes of depression over the past 5 years (for which she cannot identify a seasonal pattern) that would have met criteria for major depressive disorder. Which of the following correctly summarizes this patient's eligibility for a diagnosis of 'major depressive disorder, with seasonal pattern'? Select one: a. She does qualify for this diagnosis: the symptoms described are related to psychosocial stressors b. She does not qualify for this diagnosis: the episode must start in the fall, and the patient must have no episodes that do not have a seasonal pattern c. She does qualify for this diagnosis: the symptoms are not related to bipolar I or bipolar II disorder. d. She does not qualify for this diagnosis: the patient must have had two episodes with a seasonal relationship in the past 2 years and no non-seasonal episodes during that period

d. She does not qualify for this diagnosis: the patient must have had two episodes with a seasonal relationship in the past 2 years and no non-seasonal episodes during that period -The 'with seasonal pattern' specifier requires a regular temporal relationship between the onset of major depressive episodes (MDEs) in major depressive disorder or in bipolar I or bipolar II disorder and a particular time of the year (e.g., in the fall or winter. The diagnosis excludes cases in which there is an obvious effect of seasonal-related psychosocial stressors (e.g., regularly being unemployed every winter.) Full remissions (or a change from major depression to mania or hypomania) also occur at a characteristic time of the years (e.g., depression disappears in the spring). In the past 2 years, two MDEs must have occurred that demonstrate the temporal seasonal relationships defined above, and no non-seasonal MDEs must have occurred during that same time. Seasonal MDEs must substantially outnumber the non-seasonal MDEs that may have occurred over the individual's lifetime. The specifier 'with seasonal pattern' can be applied to the pattern of MDEs in bipolar I disorder, bipolar II disorder, or major depressive disorder, recurrent. (187-188)

A 31-year-old woman with no history of mood symptoms reports that she experiences distressing mood lability and irritability starting about 4 days before the onset of menses. She feels 'on edge,' cannot concentrate, has little enjoyment from any of her activities, and experiences bloating and swelling of her breasts. The patient reports that these symptoms started 6 months ago when she began taking oral contraceptives for the first time. If she stops the oral contraceptives and her symptoms remit, what would the diagnosis be? Select one: a. Dysthymia b. Major depressive episode c. Premenstrual syndrome d. Substance / medication induced depressive disorder e. Premenstrual dysphoric disorder

d. Substance / medication induced depressive disorder -If the woman stops the hormones and her symptoms disappear, this is consistent with substance / medication induced depressive disorder. Some women who present with moderate to severe premenstrual symptoms may be using hormonal contraceptives. If such symptoms occur after the initiation of exogenous hormone use, the symptoms may be due to the use of hormones rather than the underlying condition of PMDD.

An irritable 8-year-old child has a history of temper outbursts at home and at school. What characteristic mood feature must be also present to qualify for a diagnosis of disruptive mood regulation disorder? Select one: a. The child's mood between outburst is typically depressed. b. The child's mood between outbursts is typically hypomanic c. The mood symptoms and temper outbursts must not have persisted for more than 6 months. d. The child's mood between outbursts is typically angry or irritable e. The child's mood between outburst is typically euthymic

d. The child's mood between outbursts is typically angry or irritable -Criterion D of disruptive mood dysregulation disorder requires that the child's mood between temper outbursts be persistently irritable or angry most of the day, nearly every day, and observable by others (e.g., parents, teachers, peers). (156)

How do individuals with substance / medication induced depressive disorder differ from individuals with major depressive disorder who do not have a substance use disorder? Select one: a. They are more likely to be female b. They are more likely to have graduate school education c. They are less likely to report suicidal thoughts / attempts d. They are more likely to be male e. They are more likely to be white

d. They are more likely to be male -In a representative U.S. adult population, compared with individuals with major depressive disorder who did not have a substance use disorder, individuals with a substance-induced depressive disorder were more likely to be male, to be black, to have at most a high school diploma, to lack insurance, and to have lower family income. They were also more likely to report higher family history of substance use disorders and antisocial behavior, higher 12-month history of stressful life events, a greater number of major depressive disorder criteria, and feelings of worthlessness, insomnia / hypersomnia, and thoughts of death and suicide attempts. (179)

A 45-year-old woman with multiple sclerosis was treated with interferon beta-1a a year ago, which resolved her physical symptoms. She now presents with depressed mood (experienced daily for the past several months), middle insomnia (of recent onset), poor appetite, trouble concentrating, and lack of interest in sex. Although she has no physical symptoms, she is frequently absent from work. She denies any active plans to commit suicide but admits that she often thinks about it, as her mood has worsened. What is the most likely diagnosis? Select one: a. Persistent depressive disorder (dysthmia) and multiple sclerosis b. Major depressive disorder c. Persistent depressive disorder (dysthymia) d. Substance / medication induced depressive disorder e. Depressive disorder due to another medical condition

e. Depressive disorder due to another medical condition The essential feature of depressive disorder due to another medical condition is a prominent and persistent period of depressed mood or markedly diminished interest or pleasure in all, or almost all, activities that predominates in the clinical picture and that is thought to be related to the direct physiological effects of another medical condition. In determining whether the mood disturbance is due to another medical condition, the clinician must first establish the presence of such a condition. Furthermore, the clinician must establish that the mood disturbance is etiologically related to the other medical condition through a physiological mechanism. A careful and comprehensive assessment of multiple factors is necessary to make this judgment. (181)

A 45-year-old man is admitted to the hospital with profound hypothyroidism. He is depressed but does not meet full criteria for major depressive disorder (MDD), the diagnosis given to him by his internist. The patient has no prior history of mood disorder, and all of the depressive symptoms are temporally related to the hypothyroidism. Based on this information, you determine that a change in diagnosis - to depressive disorder due to another medical condition - is warranted, as well as a specifier to indicate that full criteria for MDD are not met. How would the full diagnosis be recorded Select one: a. Hypothyroidism would be coded on Axis III in DSM-5 b. A revision to DSM-5 is planned to deal with this. c. There is no special coding procedure in DSM-5 d. Medical disorders are not coded as part of a mental disorder diagnosis in DSM-5 e. Hypothyroidism would be recorded as the name of the 'other medical condition' in the DSM-5 diagnosis

e. Hypothyroidism would be recorded as the name of the 'other medical condition' in the DSM-5 diagnosis In recording a diagnosis of depressive disorder due to another medical condition, the name of the other medical condition is inserted in the mental disorder diagnosis (i.e., 'depressed disorder due to hypothyroidism'). In addition, the other medical condition should be coded and listed separately immediately before the depression disorder due to the medical condition. In this vignette, the full coding would be '244.9 (E03.9) hypothyroidism; 293.83 (F06.31) disorder due to hypothyroidism, with depressive features.' (The 'depressive features' specifier denotes that the full criteria are not met for a major depressive episode.). There is no longer an Axis III in DSM-5 (181).

A 26-year-old man is brought to the emergency department suffering from a sudden, severe surge of panic. He has no history of panic disorder, but he reports taking several doses of an over-the-counter cold medication earlier that day. Which of the following clinical features, if present in this case, would help to confirm a diagnosis of substance / medication-induced anxiety disorder? Select one: a. Symptoms that persist for a long time after substance / medication use b. Symptoms that are mild and do not impair functioning c. Presence of a delirium or gross confusion d. Symptoms that are in excess of what would be expected for the substance / medication e. Lack of any history of anxiety or panic symptoms

e. Lack of any history of anxiety or panic symptoms -Many substances can potentially cause anxiety symptoms, and it can sometimes be difficult to determine whether medication use is etiologically related to the onset of anxiety symptoms. Evidence to support the presence of a substance / medication-induced anxiety disorder includes a temporal association and symptoms that are consistent with the medication and dose. By definition, a substance / medication-induced anxiety disorder must cause significant distress or impairment in functioning, and it cannot occur exclusively during the course of a delirium. (226-228)

Children with disruptive mood dysregulation disorder (DMDD) often meet criteria for what additional DSM-5 diagnosis? Select one: a. Pediatric bipolar disorder b. Major depressive disorder c. Intermittent explosive disorder d. Schizophrenia e. Oppositional defiant disorder

e. Oppositional defiant disorder Because chronically irritable children and adolescents typically present with complex histories, the diagnosis of DMDD must be made while considering the presence or absence of multiple other conditions. The differential diagnosis of DMDD from both bipolar disorder and oppositional defiant disorder requires careful consideration. DMDD differs from bipolar disorder in that the former is chronic whereas the latter is episodic. DMDD differs from oppositional defiant disorder in that very severe irritability is required in the former but not the latter. For this reason, while most children who meet criteria for DMDD will also meet criteria for oppositional defiant disorder, the reverse is not the case. (158-159)

A 32 year-old woman reports sudden, unexpected episodes of intense anxiety accompanied by headaches, a rapid pulse, nausea, and shortness of breath. During the episodes she fears that she is dying, and she has presented several times to emergency departments. Each time she has been told that she is medically healthy; she is usually reassured for a time, but on the occurrence of a new episode she again becomes concerned that she has some severe medical problem. She was given lorazepam once but disliked the sedating effect and has not taken it again. She abstains from all medications and alcohol in an attempt to minimize potential causes for her attacks. What is the most likely explanation for her symptoms? Select one: a. Anxiety due to another medical condition b. Specific phobia c. Illness anxiety disorder d. Somatic symptom disorder e. Panic disorder

e. Panic disorder -The presence of sudden, unexpected panic attacks in the absence of a medical disorder is the main feature of panic disorder. In addition to worries about the attacks, many individuals report broader concerns about health and mental health outcomes. In a search for an explanation for their symptoms they may worry about having a major disease. This differs from illness anxiety disorder in that the concern with panic attacks stems from what might be seen as a reasonable concern over their dramatic and unexplained symptoms; patients with panic attacks do not show the preoccupation with having a feared disease that is typical of illness anxiety disorder (or hypochondriasis in DSM-IV). Specific phobia refers to anxiety centered on a specific trigger (212-213)

A 50 year-old man reports episodes in which he suddenly and unexpectedly awakens from sleep feeling a surge of intense fear that peaks within minutes. During this time he feels short of breath and has heart palpitations, sweating, and nausea. His medical history is significant only for hypertension, which is well controlled with hydrochlorothiazide. As a result of these symptoms he has begun to have anticipatory anxiety associated with going to sleep. What is the most likely explanation for his symptoms? Select one: a. Anxiety disorder due to another medical condition (hypertension) b. Sleep terrors c. Panic attacks d. Substance / medication-induced anxiety disorder e. Panic disorder

e. Panic disorder Panic disorder involves recurrent, unexpected panic attacks. Panic attacks are a syndrome, not a disorder, and can occur with a variety of disorders. Other medical conditions, substance-related disorders, and other psychiatric disorders must be ruled out; in this vignette, the patient's well-controlled hypertension and use of a diuretic are unlikely to be the cause of his attacks. Nocturnal panic attacks associated with sleep are an example of unexpected panic attack. Although sleep-related disorders should be ruled out, this classic presentation makes panic disorder the most likely explanation. (209-210)

When called on at school, a 7-year-old boy will only nod or write in response. The family of the child is surprised to hear this from the teacher, because the boy speaks normally when at home with his parents. The child has achieved appropriate developmental milestones, and a medical evaluation indicates that he is healthy. The boy is unable to give any explanation for his behavior, but the parents are concerned that it will affect his school performance. What diagnosis best fits this child's symptoms? Select one: a. Separation anxiety disorder b. Communication disorder c. Autism spectrum disorder d. Agoraphobia e. Selective mutism

e. Selective mutism -When encountering other individuals in social interactions, children with selective mutism do not initiate speech or reciprocally respond when spoken to. Lack of speech occurs in social interactions with children or adults. Children with selective mutism will speak in their home in the presence of immediate family members but often not even in front of close friends or second-degree relatives, such as grandparents or cousins. The disturbance is often marked by high social anxiety. Children with selective mutism often refuse to speak at school, leading to academic or educational impairment, as teachers often find it difficult to assess skills such as reading. The lack of speech may interfere with social communication, although children with this disorder sometimes use nonspoken or nonverbal means (e.g., grunting, pointing, writing) to communicate and may be willing or eager to perform or engage in social encounters when speech is not required (e.g., nonverbal parts of school plays. (195)


Kaugnay na mga set ng pag-aaral

Intro to finance chapter 6 questions, Finance Ch 6 HW

View Set

Chapter 7: Respiration and Glycolysis

View Set

Intro to Business Chapter 2 Vocabulary

View Set

Polysci chapter 3, Polysci chapter 4, Polysci chapter 5

View Set

The CE Shop Principles of Real Estate 1

View Set